Solutions to $a, b, c, frac{a}{b}+frac{b}{c}+frac{c}{a}, frac{b}{a} + frac{c}{b} + frac{a}{c} in mathbb{Z}$












9












$begingroup$


I came across a puzzle in a Maths Calendar I own. Most of them I can do fairly easily, but this one has me stumped, and I was hoping for a hint or solution. The question is:



What are the solutions to



$$left { a, b, c, dfrac{a}{b}+dfrac{b}{c}+dfrac{c}{a}, dfrac{b}{a} + dfrac{c}{b} + dfrac{a}{c} right } subset mathbb{Z}$$



I've tried a few things, but don't think I've made any meaningful progress, besides determining that $a = pm b = pm c $ are the only obvious possible solutions. My hope is to prove that no other solution can exist.





I don't know if it helps, but I also did a brute force search for coprime numbers $a,b,c$ for which $dfrac{a}{b}+dfrac{b}{c}+dfrac{c}{a} in mathbb{Z}$, with $1 leq a leq b leq c$, and $a leq 100, b leq 1000, c leq 10000$.



The reason for coprimality is that if a solution has a common factor, we can divide through by the common factor and have another solution that satisfies the conditions.



The triplets I found which satisfy this are:



$(a, b, c) = (1, 1, 1), (1,2,4), (2, 36, 81), (3, 126, 196), (4, 9, 162), (9, 14, 588), (12, 63, 98), (18, 28, 147), (98, 108, 5103)$



None of these except the first satisfy $dfrac{b}{a} + dfrac{c}{b} + dfrac{a}{c} in mathbb{Z}$.










share|cite|improve this question











$endgroup$








  • 3




    $begingroup$
    Surely, one solution is obvious.
    $endgroup$
    – Zachary Selk
    Dec 10 '18 at 13:10










  • $begingroup$
    How is $ (a,b,c)=(1,2,4) $ a solution? $ frac{b}{a} + frac{c}{b} + frac{a}{c} = frac{2}{1} + frac{4}{2} + frac{1}{4} = 4 + 1/4 $, not an integer.
    $endgroup$
    – hellHound
    Dec 10 '18 at 13:22












  • $begingroup$
    @hellhound it does not satisfy $b/a + c/b + a/c in mathbb{Z}$, only $a/b+b/c+c/a in mathbb{Z}$
    $endgroup$
    – Shakespeare
    Dec 10 '18 at 13:25










  • $begingroup$
    Oh, I noticed the line about your search now, my bad. Although, if I had to guess, none of the triples from your search except $ (1,1,1) $ would satisfy the problem's requirement.
    $endgroup$
    – hellHound
    Dec 10 '18 at 13:27








  • 1




    $begingroup$
    @hellhound correct, none of them do :)
    $endgroup$
    – Shakespeare
    Dec 10 '18 at 13:28


















9












$begingroup$


I came across a puzzle in a Maths Calendar I own. Most of them I can do fairly easily, but this one has me stumped, and I was hoping for a hint or solution. The question is:



What are the solutions to



$$left { a, b, c, dfrac{a}{b}+dfrac{b}{c}+dfrac{c}{a}, dfrac{b}{a} + dfrac{c}{b} + dfrac{a}{c} right } subset mathbb{Z}$$



I've tried a few things, but don't think I've made any meaningful progress, besides determining that $a = pm b = pm c $ are the only obvious possible solutions. My hope is to prove that no other solution can exist.





I don't know if it helps, but I also did a brute force search for coprime numbers $a,b,c$ for which $dfrac{a}{b}+dfrac{b}{c}+dfrac{c}{a} in mathbb{Z}$, with $1 leq a leq b leq c$, and $a leq 100, b leq 1000, c leq 10000$.



The reason for coprimality is that if a solution has a common factor, we can divide through by the common factor and have another solution that satisfies the conditions.



The triplets I found which satisfy this are:



$(a, b, c) = (1, 1, 1), (1,2,4), (2, 36, 81), (3, 126, 196), (4, 9, 162), (9, 14, 588), (12, 63, 98), (18, 28, 147), (98, 108, 5103)$



None of these except the first satisfy $dfrac{b}{a} + dfrac{c}{b} + dfrac{a}{c} in mathbb{Z}$.










share|cite|improve this question











$endgroup$








  • 3




    $begingroup$
    Surely, one solution is obvious.
    $endgroup$
    – Zachary Selk
    Dec 10 '18 at 13:10










  • $begingroup$
    How is $ (a,b,c)=(1,2,4) $ a solution? $ frac{b}{a} + frac{c}{b} + frac{a}{c} = frac{2}{1} + frac{4}{2} + frac{1}{4} = 4 + 1/4 $, not an integer.
    $endgroup$
    – hellHound
    Dec 10 '18 at 13:22












  • $begingroup$
    @hellhound it does not satisfy $b/a + c/b + a/c in mathbb{Z}$, only $a/b+b/c+c/a in mathbb{Z}$
    $endgroup$
    – Shakespeare
    Dec 10 '18 at 13:25










  • $begingroup$
    Oh, I noticed the line about your search now, my bad. Although, if I had to guess, none of the triples from your search except $ (1,1,1) $ would satisfy the problem's requirement.
    $endgroup$
    – hellHound
    Dec 10 '18 at 13:27








  • 1




    $begingroup$
    @hellhound correct, none of them do :)
    $endgroup$
    – Shakespeare
    Dec 10 '18 at 13:28
















9












9








9


7



$begingroup$


I came across a puzzle in a Maths Calendar I own. Most of them I can do fairly easily, but this one has me stumped, and I was hoping for a hint or solution. The question is:



What are the solutions to



$$left { a, b, c, dfrac{a}{b}+dfrac{b}{c}+dfrac{c}{a}, dfrac{b}{a} + dfrac{c}{b} + dfrac{a}{c} right } subset mathbb{Z}$$



I've tried a few things, but don't think I've made any meaningful progress, besides determining that $a = pm b = pm c $ are the only obvious possible solutions. My hope is to prove that no other solution can exist.





I don't know if it helps, but I also did a brute force search for coprime numbers $a,b,c$ for which $dfrac{a}{b}+dfrac{b}{c}+dfrac{c}{a} in mathbb{Z}$, with $1 leq a leq b leq c$, and $a leq 100, b leq 1000, c leq 10000$.



The reason for coprimality is that if a solution has a common factor, we can divide through by the common factor and have another solution that satisfies the conditions.



The triplets I found which satisfy this are:



$(a, b, c) = (1, 1, 1), (1,2,4), (2, 36, 81), (3, 126, 196), (4, 9, 162), (9, 14, 588), (12, 63, 98), (18, 28, 147), (98, 108, 5103)$



None of these except the first satisfy $dfrac{b}{a} + dfrac{c}{b} + dfrac{a}{c} in mathbb{Z}$.










share|cite|improve this question











$endgroup$




I came across a puzzle in a Maths Calendar I own. Most of them I can do fairly easily, but this one has me stumped, and I was hoping for a hint or solution. The question is:



What are the solutions to



$$left { a, b, c, dfrac{a}{b}+dfrac{b}{c}+dfrac{c}{a}, dfrac{b}{a} + dfrac{c}{b} + dfrac{a}{c} right } subset mathbb{Z}$$



I've tried a few things, but don't think I've made any meaningful progress, besides determining that $a = pm b = pm c $ are the only obvious possible solutions. My hope is to prove that no other solution can exist.





I don't know if it helps, but I also did a brute force search for coprime numbers $a,b,c$ for which $dfrac{a}{b}+dfrac{b}{c}+dfrac{c}{a} in mathbb{Z}$, with $1 leq a leq b leq c$, and $a leq 100, b leq 1000, c leq 10000$.



The reason for coprimality is that if a solution has a common factor, we can divide through by the common factor and have another solution that satisfies the conditions.



The triplets I found which satisfy this are:



$(a, b, c) = (1, 1, 1), (1,2,4), (2, 36, 81), (3, 126, 196), (4, 9, 162), (9, 14, 588), (12, 63, 98), (18, 28, 147), (98, 108, 5103)$



None of these except the first satisfy $dfrac{b}{a} + dfrac{c}{b} + dfrac{a}{c} in mathbb{Z}$.







elementary-number-theory divisibility diophantine-equations recreational-mathematics problem-solving






share|cite|improve this question















share|cite|improve this question













share|cite|improve this question




share|cite|improve this question








edited Dec 11 '18 at 12:44









Batominovski

1




1










asked Dec 10 '18 at 13:08









ShakespeareShakespeare

2,504923




2,504923








  • 3




    $begingroup$
    Surely, one solution is obvious.
    $endgroup$
    – Zachary Selk
    Dec 10 '18 at 13:10










  • $begingroup$
    How is $ (a,b,c)=(1,2,4) $ a solution? $ frac{b}{a} + frac{c}{b} + frac{a}{c} = frac{2}{1} + frac{4}{2} + frac{1}{4} = 4 + 1/4 $, not an integer.
    $endgroup$
    – hellHound
    Dec 10 '18 at 13:22












  • $begingroup$
    @hellhound it does not satisfy $b/a + c/b + a/c in mathbb{Z}$, only $a/b+b/c+c/a in mathbb{Z}$
    $endgroup$
    – Shakespeare
    Dec 10 '18 at 13:25










  • $begingroup$
    Oh, I noticed the line about your search now, my bad. Although, if I had to guess, none of the triples from your search except $ (1,1,1) $ would satisfy the problem's requirement.
    $endgroup$
    – hellHound
    Dec 10 '18 at 13:27








  • 1




    $begingroup$
    @hellhound correct, none of them do :)
    $endgroup$
    – Shakespeare
    Dec 10 '18 at 13:28
















  • 3




    $begingroup$
    Surely, one solution is obvious.
    $endgroup$
    – Zachary Selk
    Dec 10 '18 at 13:10










  • $begingroup$
    How is $ (a,b,c)=(1,2,4) $ a solution? $ frac{b}{a} + frac{c}{b} + frac{a}{c} = frac{2}{1} + frac{4}{2} + frac{1}{4} = 4 + 1/4 $, not an integer.
    $endgroup$
    – hellHound
    Dec 10 '18 at 13:22












  • $begingroup$
    @hellhound it does not satisfy $b/a + c/b + a/c in mathbb{Z}$, only $a/b+b/c+c/a in mathbb{Z}$
    $endgroup$
    – Shakespeare
    Dec 10 '18 at 13:25










  • $begingroup$
    Oh, I noticed the line about your search now, my bad. Although, if I had to guess, none of the triples from your search except $ (1,1,1) $ would satisfy the problem's requirement.
    $endgroup$
    – hellHound
    Dec 10 '18 at 13:27








  • 1




    $begingroup$
    @hellhound correct, none of them do :)
    $endgroup$
    – Shakespeare
    Dec 10 '18 at 13:28










3




3




$begingroup$
Surely, one solution is obvious.
$endgroup$
– Zachary Selk
Dec 10 '18 at 13:10




$begingroup$
Surely, one solution is obvious.
$endgroup$
– Zachary Selk
Dec 10 '18 at 13:10












$begingroup$
How is $ (a,b,c)=(1,2,4) $ a solution? $ frac{b}{a} + frac{c}{b} + frac{a}{c} = frac{2}{1} + frac{4}{2} + frac{1}{4} = 4 + 1/4 $, not an integer.
$endgroup$
– hellHound
Dec 10 '18 at 13:22






$begingroup$
How is $ (a,b,c)=(1,2,4) $ a solution? $ frac{b}{a} + frac{c}{b} + frac{a}{c} = frac{2}{1} + frac{4}{2} + frac{1}{4} = 4 + 1/4 $, not an integer.
$endgroup$
– hellHound
Dec 10 '18 at 13:22














$begingroup$
@hellhound it does not satisfy $b/a + c/b + a/c in mathbb{Z}$, only $a/b+b/c+c/a in mathbb{Z}$
$endgroup$
– Shakespeare
Dec 10 '18 at 13:25




$begingroup$
@hellhound it does not satisfy $b/a + c/b + a/c in mathbb{Z}$, only $a/b+b/c+c/a in mathbb{Z}$
$endgroup$
– Shakespeare
Dec 10 '18 at 13:25












$begingroup$
Oh, I noticed the line about your search now, my bad. Although, if I had to guess, none of the triples from your search except $ (1,1,1) $ would satisfy the problem's requirement.
$endgroup$
– hellHound
Dec 10 '18 at 13:27






$begingroup$
Oh, I noticed the line about your search now, my bad. Although, if I had to guess, none of the triples from your search except $ (1,1,1) $ would satisfy the problem's requirement.
$endgroup$
– hellHound
Dec 10 '18 at 13:27






1




1




$begingroup$
@hellhound correct, none of them do :)
$endgroup$
– Shakespeare
Dec 10 '18 at 13:28






$begingroup$
@hellhound correct, none of them do :)
$endgroup$
– Shakespeare
Dec 10 '18 at 13:28












2 Answers
2






active

oldest

votes


















16












$begingroup$

Suppose that $displaystyle a,b,c,frac{a}{b}+frac{b}{c}+frac{c}{a},frac{a}{c}+frac{b}{a}+frac{c}{b} in mathbb Z$.
Consider polynomial
$$P(x)=left(x-frac{a}{b}right)left(x-frac{b}{c}right)left(x-frac{c}{a}right) = x^3-left(frac{a}{b}+frac{b}{c}+frac{c}{a}right)x^2+left(frac{a}{c}+frac{b}{a}+frac{c}{b}right)x-1.$$
Its coefficients are integers. Since the leading coefficient is $1$, all rational roots of $P$ are integers. Since the constant term is $-1$, it follows that all integer roots of $P$ are $1$ or $-1$ (they must divide the constant term). Since $dfrac ab, dfrac bc, dfrac ca$ are rational roots of $P$, it follows that $dfrac ab, dfrac bc, dfrac ca in {-1,1}$.






share|cite|improve this answer











$endgroup$













  • $begingroup$
    Beautiful! I don't think I ever would have thought of this.
    $endgroup$
    – Shakespeare
    Dec 10 '18 at 15:27










  • $begingroup$
    @Shakespeare See also this prior symmetric variant.
    $endgroup$
    – Bill Dubuque
    Dec 10 '18 at 22:21



















1












$begingroup$

Let $(a,b,c)$ satisfy the requirements. Let $(a,b,c)$ are coprime. Then
$abc$ divides both
$a^2c + b^2a + c^2b$ and $a^2b+b^2c+c^2a$.



Let $p$ be a prime factor of $a$.
Let $d$ be the largest number such that $p^d$ divides $a$.
Then $p$ divides $b^2c$ and $c^2b$. Assume $p$ divides $b$ (and does not divide $c$).



Since $p^{d+1}$ divides $a^2$, $ab$, and $abc$, where the latter divides $a^2c + b^2a + c^2b$, it follows $p^{d+1}$ divides $b$.
This in turn implies that $p^{d+1}$ divides $a^2b+b^2c+c^2a$.
Now, $p$ does not divide $c$ by assumption of coprimality, hence $p^{d+1}$ divides $a$, a contradiction to the maximality of $d$.



Hence, none of $a,b,c$ has a prime factor. So all these numbers are equal $pm1$.






share|cite|improve this answer











$endgroup$













    Your Answer





    StackExchange.ifUsing("editor", function () {
    return StackExchange.using("mathjaxEditing", function () {
    StackExchange.MarkdownEditor.creationCallbacks.add(function (editor, postfix) {
    StackExchange.mathjaxEditing.prepareWmdForMathJax(editor, postfix, [["$", "$"], ["\\(","\\)"]]);
    });
    });
    }, "mathjax-editing");

    StackExchange.ready(function() {
    var channelOptions = {
    tags: "".split(" "),
    id: "69"
    };
    initTagRenderer("".split(" "), "".split(" "), channelOptions);

    StackExchange.using("externalEditor", function() {
    // Have to fire editor after snippets, if snippets enabled
    if (StackExchange.settings.snippets.snippetsEnabled) {
    StackExchange.using("snippets", function() {
    createEditor();
    });
    }
    else {
    createEditor();
    }
    });

    function createEditor() {
    StackExchange.prepareEditor({
    heartbeatType: 'answer',
    autoActivateHeartbeat: false,
    convertImagesToLinks: true,
    noModals: true,
    showLowRepImageUploadWarning: true,
    reputationToPostImages: 10,
    bindNavPrevention: true,
    postfix: "",
    imageUploader: {
    brandingHtml: "Powered by u003ca class="icon-imgur-white" href="https://imgur.com/"u003eu003c/au003e",
    contentPolicyHtml: "User contributions licensed under u003ca href="https://creativecommons.org/licenses/by-sa/3.0/"u003ecc by-sa 3.0 with attribution requiredu003c/au003e u003ca href="https://stackoverflow.com/legal/content-policy"u003e(content policy)u003c/au003e",
    allowUrls: true
    },
    noCode: true, onDemand: true,
    discardSelector: ".discard-answer"
    ,immediatelyShowMarkdownHelp:true
    });


    }
    });














    draft saved

    draft discarded


















    StackExchange.ready(
    function () {
    StackExchange.openid.initPostLogin('.new-post-login', 'https%3a%2f%2fmath.stackexchange.com%2fquestions%2f3033895%2fsolutions-to-a-b-c-fracab-fracbc-fracca-fracba-f%23new-answer', 'question_page');
    }
    );

    Post as a guest















    Required, but never shown

























    2 Answers
    2






    active

    oldest

    votes








    2 Answers
    2






    active

    oldest

    votes









    active

    oldest

    votes






    active

    oldest

    votes









    16












    $begingroup$

    Suppose that $displaystyle a,b,c,frac{a}{b}+frac{b}{c}+frac{c}{a},frac{a}{c}+frac{b}{a}+frac{c}{b} in mathbb Z$.
    Consider polynomial
    $$P(x)=left(x-frac{a}{b}right)left(x-frac{b}{c}right)left(x-frac{c}{a}right) = x^3-left(frac{a}{b}+frac{b}{c}+frac{c}{a}right)x^2+left(frac{a}{c}+frac{b}{a}+frac{c}{b}right)x-1.$$
    Its coefficients are integers. Since the leading coefficient is $1$, all rational roots of $P$ are integers. Since the constant term is $-1$, it follows that all integer roots of $P$ are $1$ or $-1$ (they must divide the constant term). Since $dfrac ab, dfrac bc, dfrac ca$ are rational roots of $P$, it follows that $dfrac ab, dfrac bc, dfrac ca in {-1,1}$.






    share|cite|improve this answer











    $endgroup$













    • $begingroup$
      Beautiful! I don't think I ever would have thought of this.
      $endgroup$
      – Shakespeare
      Dec 10 '18 at 15:27










    • $begingroup$
      @Shakespeare See also this prior symmetric variant.
      $endgroup$
      – Bill Dubuque
      Dec 10 '18 at 22:21
















    16












    $begingroup$

    Suppose that $displaystyle a,b,c,frac{a}{b}+frac{b}{c}+frac{c}{a},frac{a}{c}+frac{b}{a}+frac{c}{b} in mathbb Z$.
    Consider polynomial
    $$P(x)=left(x-frac{a}{b}right)left(x-frac{b}{c}right)left(x-frac{c}{a}right) = x^3-left(frac{a}{b}+frac{b}{c}+frac{c}{a}right)x^2+left(frac{a}{c}+frac{b}{a}+frac{c}{b}right)x-1.$$
    Its coefficients are integers. Since the leading coefficient is $1$, all rational roots of $P$ are integers. Since the constant term is $-1$, it follows that all integer roots of $P$ are $1$ or $-1$ (they must divide the constant term). Since $dfrac ab, dfrac bc, dfrac ca$ are rational roots of $P$, it follows that $dfrac ab, dfrac bc, dfrac ca in {-1,1}$.






    share|cite|improve this answer











    $endgroup$













    • $begingroup$
      Beautiful! I don't think I ever would have thought of this.
      $endgroup$
      – Shakespeare
      Dec 10 '18 at 15:27










    • $begingroup$
      @Shakespeare See also this prior symmetric variant.
      $endgroup$
      – Bill Dubuque
      Dec 10 '18 at 22:21














    16












    16








    16





    $begingroup$

    Suppose that $displaystyle a,b,c,frac{a}{b}+frac{b}{c}+frac{c}{a},frac{a}{c}+frac{b}{a}+frac{c}{b} in mathbb Z$.
    Consider polynomial
    $$P(x)=left(x-frac{a}{b}right)left(x-frac{b}{c}right)left(x-frac{c}{a}right) = x^3-left(frac{a}{b}+frac{b}{c}+frac{c}{a}right)x^2+left(frac{a}{c}+frac{b}{a}+frac{c}{b}right)x-1.$$
    Its coefficients are integers. Since the leading coefficient is $1$, all rational roots of $P$ are integers. Since the constant term is $-1$, it follows that all integer roots of $P$ are $1$ or $-1$ (they must divide the constant term). Since $dfrac ab, dfrac bc, dfrac ca$ are rational roots of $P$, it follows that $dfrac ab, dfrac bc, dfrac ca in {-1,1}$.






    share|cite|improve this answer











    $endgroup$



    Suppose that $displaystyle a,b,c,frac{a}{b}+frac{b}{c}+frac{c}{a},frac{a}{c}+frac{b}{a}+frac{c}{b} in mathbb Z$.
    Consider polynomial
    $$P(x)=left(x-frac{a}{b}right)left(x-frac{b}{c}right)left(x-frac{c}{a}right) = x^3-left(frac{a}{b}+frac{b}{c}+frac{c}{a}right)x^2+left(frac{a}{c}+frac{b}{a}+frac{c}{b}right)x-1.$$
    Its coefficients are integers. Since the leading coefficient is $1$, all rational roots of $P$ are integers. Since the constant term is $-1$, it follows that all integer roots of $P$ are $1$ or $-1$ (they must divide the constant term). Since $dfrac ab, dfrac bc, dfrac ca$ are rational roots of $P$, it follows that $dfrac ab, dfrac bc, dfrac ca in {-1,1}$.







    share|cite|improve this answer














    share|cite|improve this answer



    share|cite|improve this answer








    edited Dec 10 '18 at 16:34

























    answered Dec 10 '18 at 15:13









    timon92timon92

    4,3431826




    4,3431826












    • $begingroup$
      Beautiful! I don't think I ever would have thought of this.
      $endgroup$
      – Shakespeare
      Dec 10 '18 at 15:27










    • $begingroup$
      @Shakespeare See also this prior symmetric variant.
      $endgroup$
      – Bill Dubuque
      Dec 10 '18 at 22:21


















    • $begingroup$
      Beautiful! I don't think I ever would have thought of this.
      $endgroup$
      – Shakespeare
      Dec 10 '18 at 15:27










    • $begingroup$
      @Shakespeare See also this prior symmetric variant.
      $endgroup$
      – Bill Dubuque
      Dec 10 '18 at 22:21
















    $begingroup$
    Beautiful! I don't think I ever would have thought of this.
    $endgroup$
    – Shakespeare
    Dec 10 '18 at 15:27




    $begingroup$
    Beautiful! I don't think I ever would have thought of this.
    $endgroup$
    – Shakespeare
    Dec 10 '18 at 15:27












    $begingroup$
    @Shakespeare See also this prior symmetric variant.
    $endgroup$
    – Bill Dubuque
    Dec 10 '18 at 22:21




    $begingroup$
    @Shakespeare See also this prior symmetric variant.
    $endgroup$
    – Bill Dubuque
    Dec 10 '18 at 22:21











    1












    $begingroup$

    Let $(a,b,c)$ satisfy the requirements. Let $(a,b,c)$ are coprime. Then
    $abc$ divides both
    $a^2c + b^2a + c^2b$ and $a^2b+b^2c+c^2a$.



    Let $p$ be a prime factor of $a$.
    Let $d$ be the largest number such that $p^d$ divides $a$.
    Then $p$ divides $b^2c$ and $c^2b$. Assume $p$ divides $b$ (and does not divide $c$).



    Since $p^{d+1}$ divides $a^2$, $ab$, and $abc$, where the latter divides $a^2c + b^2a + c^2b$, it follows $p^{d+1}$ divides $b$.
    This in turn implies that $p^{d+1}$ divides $a^2b+b^2c+c^2a$.
    Now, $p$ does not divide $c$ by assumption of coprimality, hence $p^{d+1}$ divides $a$, a contradiction to the maximality of $d$.



    Hence, none of $a,b,c$ has a prime factor. So all these numbers are equal $pm1$.






    share|cite|improve this answer











    $endgroup$


















      1












      $begingroup$

      Let $(a,b,c)$ satisfy the requirements. Let $(a,b,c)$ are coprime. Then
      $abc$ divides both
      $a^2c + b^2a + c^2b$ and $a^2b+b^2c+c^2a$.



      Let $p$ be a prime factor of $a$.
      Let $d$ be the largest number such that $p^d$ divides $a$.
      Then $p$ divides $b^2c$ and $c^2b$. Assume $p$ divides $b$ (and does not divide $c$).



      Since $p^{d+1}$ divides $a^2$, $ab$, and $abc$, where the latter divides $a^2c + b^2a + c^2b$, it follows $p^{d+1}$ divides $b$.
      This in turn implies that $p^{d+1}$ divides $a^2b+b^2c+c^2a$.
      Now, $p$ does not divide $c$ by assumption of coprimality, hence $p^{d+1}$ divides $a$, a contradiction to the maximality of $d$.



      Hence, none of $a,b,c$ has a prime factor. So all these numbers are equal $pm1$.






      share|cite|improve this answer











      $endgroup$
















        1












        1








        1





        $begingroup$

        Let $(a,b,c)$ satisfy the requirements. Let $(a,b,c)$ are coprime. Then
        $abc$ divides both
        $a^2c + b^2a + c^2b$ and $a^2b+b^2c+c^2a$.



        Let $p$ be a prime factor of $a$.
        Let $d$ be the largest number such that $p^d$ divides $a$.
        Then $p$ divides $b^2c$ and $c^2b$. Assume $p$ divides $b$ (and does not divide $c$).



        Since $p^{d+1}$ divides $a^2$, $ab$, and $abc$, where the latter divides $a^2c + b^2a + c^2b$, it follows $p^{d+1}$ divides $b$.
        This in turn implies that $p^{d+1}$ divides $a^2b+b^2c+c^2a$.
        Now, $p$ does not divide $c$ by assumption of coprimality, hence $p^{d+1}$ divides $a$, a contradiction to the maximality of $d$.



        Hence, none of $a,b,c$ has a prime factor. So all these numbers are equal $pm1$.






        share|cite|improve this answer











        $endgroup$



        Let $(a,b,c)$ satisfy the requirements. Let $(a,b,c)$ are coprime. Then
        $abc$ divides both
        $a^2c + b^2a + c^2b$ and $a^2b+b^2c+c^2a$.



        Let $p$ be a prime factor of $a$.
        Let $d$ be the largest number such that $p^d$ divides $a$.
        Then $p$ divides $b^2c$ and $c^2b$. Assume $p$ divides $b$ (and does not divide $c$).



        Since $p^{d+1}$ divides $a^2$, $ab$, and $abc$, where the latter divides $a^2c + b^2a + c^2b$, it follows $p^{d+1}$ divides $b$.
        This in turn implies that $p^{d+1}$ divides $a^2b+b^2c+c^2a$.
        Now, $p$ does not divide $c$ by assumption of coprimality, hence $p^{d+1}$ divides $a$, a contradiction to the maximality of $d$.



        Hence, none of $a,b,c$ has a prime factor. So all these numbers are equal $pm1$.







        share|cite|improve this answer














        share|cite|improve this answer



        share|cite|improve this answer








        edited Dec 10 '18 at 15:15

























        answered Dec 10 '18 at 15:10









        dawdaw

        24.1k1544




        24.1k1544






























            draft saved

            draft discarded




















































            Thanks for contributing an answer to Mathematics Stack Exchange!


            • Please be sure to answer the question. Provide details and share your research!

            But avoid



            • Asking for help, clarification, or responding to other answers.

            • Making statements based on opinion; back them up with references or personal experience.


            Use MathJax to format equations. MathJax reference.


            To learn more, see our tips on writing great answers.




            draft saved


            draft discarded














            StackExchange.ready(
            function () {
            StackExchange.openid.initPostLogin('.new-post-login', 'https%3a%2f%2fmath.stackexchange.com%2fquestions%2f3033895%2fsolutions-to-a-b-c-fracab-fracbc-fracca-fracba-f%23new-answer', 'question_page');
            }
            );

            Post as a guest















            Required, but never shown





















































            Required, but never shown














            Required, but never shown












            Required, but never shown







            Required, but never shown

































            Required, but never shown














            Required, but never shown












            Required, but never shown







            Required, but never shown







            Popular posts from this blog

            Basket-ball féminin

            Different font size/position of beamer's navigation symbols template's content depending on regular/plain...

            I want to find a topological embedding $f : X rightarrow Y$ and $g: Y rightarrow X$, yet $X$ is not...